Oct 31

早上整理了下,希望没有错误。其中非数学类的暂时缺少第四题,试卷见下面的链接。

第五届大学生数学竞赛试卷(数学类,2013)

第五届大学生数学竞赛试卷(非数学类,2013)

Oct 13

\[ f(a)=\int_{0}^{+\infty}\frac{e^{-x}}{|\sin x|^{a}}dx \]
可以看到,当$a\leq 0$时是一致收敛的,$a\geq 1$时是发散的,下证明在$(0,1)$内闭一致收敛。
将积分区域分割使无穷积分变成个级数,再做变量替换,得到
\begin{align}
f(a)&=\sum_{k=0}^{\infty}\int_{k\pi}^{(k+1)\pi}\frac{e^{-x}}{|\sin x|^a}dx\qquad (x=k\pi+t) \\
&=\sum_{k=0}^{\infty}e^{-k\pi}\int_{0}^{\pi}\frac{e^{-t}}{\sin^{a}{t}}dt=\frac{1}{1-e^{-\pi}}\int_{0}^{\pi}\frac{e^{-t}}{\sin^{a}{t}}dt  \end{align}
注意到不等式(对$0<t\leq 1,0<\varepsilon<\frac{1}{2},\varepsilon\leq a\leq 1-\varepsilon$)
\[ \frac{e^{-t}}{\sin^{a}{t}}\leq \left(\frac{\pi}{2t}\right)^{a}\leq \left(\frac{\pi}{2}\right)^{1-\varepsilon}\cdot\frac{1}{t^{1-\varepsilon}}\]
所以,根据Weierstrass判别法,可知积分
\[ \int_{0}^{1}\frac{e^{-t}}{\sin^{a}{t}}dt \]
关于$a\in[\varepsilon,1-\varepsilon]$一致收敛,同理,积分
\[  \int_{1}^{\pi}\frac{e^{-t}}{\sin^{a}{t}}dt \]
也在$[\varepsilon,1-\varepsilon]$一致收敛。

 

__________________________________________________________________________________

计算
\[ I(\alpha)=\int_{0}^{+\infty}\frac{\ln(\alpha^2+x^2)}{\beta^2+x^2}dx \]
首先讨论$\beta\neq 0$的情况,记$f(x,\alpha)=\frac{\ln(\alpha^2+x^2)}{\beta^2+x^2}$,则
\[ f'_{\alpha}(x,\alpha)=\frac{2\alpha}{(\alpha^2+x^2)(\beta^2+x^2)} \]
我们看到$f$与$f'_{\alpha}$在$(0,+\infty)\times(-\infty,+\infty)$上连续。注意到
\[ \frac{|\ln(\alpha^2+x^2)|}{\beta^2+x^2}\leq \frac{\phi(x)}{\beta^2+x^2}\qquad (\phi(x)=|\ln(A^2+x^2)| \]
可知积分$I(\alpha)$在任一区间$[-A,A]$上一致收敛,另外
\[ \frac{2\alpha}{(\alpha^2+x^2)(\beta^2+x^2)}\leq \frac{2A}{(\varepsilon^2+x^2)(\beta^2+x^2)} \]
知积分
\[ I'(\alpha)=\int_{0}^{+\infty}\frac{2\alpha dx}{(\alpha^2+x^2)(\beta^2+x^2)} \]
在$0<\varepsilon\leq |\alpha|\leq A$上一致收敛。从而
\[ I'(\alpha)=\frac{\pi\alpha}{|\alpha\beta|(|\alpha|+|\beta|)}\qquad (\alpha\beta\neq 0) \]
\[ I(\alpha)=\frac{\pi\ln(|\alpha|+|\beta|)}{|\beta|}+C \]

\begin{align}
I(0)&=2\int_{0}^{+\infty}\frac{\ln{x}}{\beta^2+x^2}dx=\frac{2}{|\beta|}\int_{0}^{+\infty}+\frac{2}{|\beta|}\int_{0}^{\infty}\frac{\ln t}{1+t^2}dt\\
&=\pi\frac{\ln|\beta|}{|\beta|}
\end{align}
这说明$C=0$
\[ I(\alpha)=\frac{\pi\ln(|\alpha|+|\beta|)}{|\beta|} \]
若$\beta=0$,则积分$I(\alpha)$只在$|\alpha|=1$时收敛,此时,有
\[ I(\alpha)=\int_{0}^{\infty}\frac{\ln(1+x^2)}{x^2}dx=\pi \]

 

Sep 22

计算
\[ \sum_{k=1}^{\infty}\frac{\Gamma^2(k)}{k\Gamma(2k)} \]
我们看到级数就是
\[ \sum_{k=1}^{\infty}\frac{[(k-1)!]^2}{k\cdot (2k)!} \]
我们从一个显然的公式说起
\[ \frac{1}{1+x^2}=\sum_{k=0}^{\infty}(-1)^k x^{2k} \]
两边对$x$积分,得到
\[ \arctan x=\sum_{n=0}^{+\infty}\frac{(-1)^n x^{2n+1}}{2n+1} \]
注意到有
\[ \arctan\left(\frac{x}{\sqrt{1-x^2}}\right)=\arcsin{x} \]
我们用$ \left(\frac{x}{\sqrt{1-x^2}}\right)$替换$x$得到
\[ \arcsin{x}=\sum_{n=0}^{+\infty}(-1)^n \frac{1}{2n+1}\left(\frac{x}{\sqrt{1-x^2}}\right)^{2n+1}\]
\begin{align*}
\frac{x\arcsin{x}}{\sqrt{1-x^2}}&=\sum_{n=1}^{\infty}\frac{(-1)^{n-1}x^{2n}}{(2n-1)(1-x^2)^n}\\
&=\sum_{n=1}^{\infty}\frac{(-1)^{n-1}}{2n-1}\sum_{j=0}^{\infty}C_{n+j-1}^{j}x^{2(j+n)}\\
&=\sum_{m=1}^{\infty}x^{2m}\sum_{k=1}^{m}\frac{(-1)^{k-1}(m-1)!}{(k-1)!(m-k)!(2k-1)} 
\end{align*}
这里用到了
\[ \frac{1}{(1-x)^n}=\sum_{m=0}^{\infty}\frac{(m+n-1)!}{m!\cdot (n-1)!}x^{m} \]
注意到恒等式
\[ m\cdot \text{C}_{2m}^{m}\sum_{j=0}^{m-1}\frac{(-1)^{j}(m-1)!}{j!(m-j-1)!(2j+1)}=2^{2m-1} \]
我们得到
\[ \frac{2x\arcsin{x}}{\sqrt{1-x^2}}=\sum_{n=1}^{\infty}\frac{(2x)^{2n}}{n\cdot \text{C}_{2n}^{n}} \]
两边除$x$后积分,得到
\[ (\arcsin{x})^2=\frac{1}{2}\sum_{n=1}^{\infty}\frac{(2x)^{2n}}{n^2 \text{C}_{2n}^{n}} \]
\[ 4(\arcsin{x})^2=\sum_{n=1}^{\infty}\frac{[(n-1)!]^2}{n\cdot (2n-1)!}\cdot (2x)^{2n} \]
令$x=\frac{1}{2}$
得到级数
\[ \sum_{k=1}^{\infty}\frac{\Gamma^2(k)}{k\Gamma(2k)}=\frac{\pi^2}{9} \]

Aug 31

设$f(x)$在$[0,2]$内连续,且在$(0,2)$内可导,且$f(1)=0$,求证:$\exists \xi\in(0,2)$,使得
\[ f'(\xi)=\frac{\pi(\xi-\tan \xi)}{2\xi^2\sec \xi-\pi\xi\tan \xi}f(\xi) \]
(西西)


证明 构造
\[ F(x)=\left(2-\pi\cdot\frac{\sin{x}}{x}\right)f(x) \]
由于$F\left(\frac{\pi}{2}\right)=F(1)=0 $
由Rolle定理知$\exists \xi\in(0,2)$使得
\[ F'(\xi)=0 \]
\[ F'(x)=\pi\left(\frac{\sin{x}-x\cos{x}}{x^2}\right)f(x)+f'(x)\left(\frac{2x-\pi\sin{x}}{x}\right)\]
\[ \pi\left(\frac{\sin{\xi}-\xi\cos{\xi}}{\xi^2}\right)f(\xi)+f'(\xi)\left(\frac{2\xi-\pi\sin{\xi}}{\xi}\right)=0\]
\[ \Rightarrow f'(x)=\frac{\pi(\xi-\tan \xi)}{2\xi^2\sec \xi-\pi\xi\tan \xi}f(\xi) \]
 

Aug 27

Prove that

\begin{equation}
\int_{-\infty}^{\infty} \frac{\ln(1+e^{ax})}{1+e^{bx}}\, dx = \frac{\pi^{2}}{12} \frac{a^{2} + b^{2}}{a b^{2}}.
\end{equation}

(sos440)

Proof:
Let $I$ denote the integral  . With the substitution $t=e^{bx}$, we have
\begin{align*}
I&= \int_{-\infty}^{\infty} \frac{\ln(1+e^{ax})}{1+e^{bx}}\, dx
 = \frac{1}{b} \int_{0}^{\infty} \frac{\ln(1+t^{\frac{a}{b}})}{t(1+t)}\, dt.
\end{align*}
Then with the substitution $t\mapsto\frac{1}{t}$, it follows that
\begin{align*}
I&= \frac{1}{b} \int_{0}^{\infty} \frac{\ln(1+t^{\frac{a}{b}}) - \frac{a}{b} \ln t}{1+t}\, dt.
\end{align*}
Summing two identities above, we have

\begin{align*}
2I&= \frac{1}{b} \int_{0}^{\infty} \left( \frac{\ln(1 + t^{\frac{a}{b}})}{t} - \frac{a}{b} \frac{\ln t}{1 + t} \right) \, dt \\
&= \lim_{R\to\infty} \frac{1}{b} \int_{0}^{R} \left( \frac{\ln(1 + t^{\frac{a}{b}})}{t} - \frac{a}{b} \frac{\ln t}{1 + t} \right) \, dt.
\end{align*}
Now we focus on the integral inside the limit. Simplifying, we have
\begin{align*}
& \frac{1}{b} \int_{0}^{R} \left( \frac{\ln(1 + t^{\frac{a}{b}})}{t} - \frac{a}{b} \frac{\ln t}{1 + t} \right) \, dt \\
&= \frac{1}{a} \int_{0}^{R^{\frac{a}{b}}} \frac{\ln(1 + t)}{t} \, dt - \frac{a}{b^{2}} \int_{0}^{R} \frac{\ln t}{1 + t} \, dt \\
&= \frac{1}{a} \int_{0}^{R^{\frac{a}{b}}} \frac{\ln(1 + t)}{t} \, dt - \frac{a}{b^{2}} \ln R \ln (R+1) + \frac{a}{b^{2}} \int_{0}^{R} \frac{\ln (1+t)}{t} \, dt \\
&= -\frac{1}{a} \mathrm{Li}_{2}(-R^{\frac{a}{b}}) - \frac{a}{b^{2}} \mathrm{Li}_{2} (-R) - \frac{a}{b^{2}} \ln R \ln (R+1).
\end{align*}
But we know that the dilogarithm satisfies
\begin{align*}
-\mathrm{Li}_{2}(-x)
&= \mathrm{Li}_{2}\left( \frac{x}{1+x} \right) + \frac{1}{2}\ln^{2}(x + 1)
= \zeta(2) + \frac{1}{2}\ln^2 x + o(1)
\end{align*}
as $x\to\infty$ . Plugging this identity above, it follows that
\begin{align*}
\frac{1}{b} \int_{0}^{R} \left( \frac{\ln(1 + t^{\frac{a}{b}})}{t} - \frac{a}{b} \frac{\ln t}{1 + t} \right) \, dt
&= \frac{a^2 + b^2}{a b^2} \zeta(2) + o(1)
\end{align*}
as $R\to\infty$, and therefore the proof is complete.